Đến nội dung

Visitor nội dung

Có 63 mục bởi Visitor (Tìm giới hạn từ 30-04-2020)



Sắp theo                Sắp xếp  

#647158 Tìm $a,b,c$ là các số nguyên dương thỏa mãn $(a,b,c)=1$ ,...

Đã gửi bởi Visitor on 30-07-2016 - 10:01 trong Số học

Tìm $a,b,c$ là các số nguyên dương thỏa mãn $(a,b,c)=1$ , $a^2+b^2=c^2$, $a^2=b+c$

Từ gt suy ra $c^2-b^2=b+c\Rightarrow c-b=1\Rightarrow c=b+1$

thay lại vào đề ta được $a^2=2b+1$.

Vậy các bộ a,b,c thỏa đề là : $t,\frac{t^2-1}{2},\frac{t^2+1}{2}$ với $t$ nguyên dương lẻ lớn hơn 1.




#637500 Marathon số học Olympic

Đã gửi bởi Visitor on 01-06-2016 - 22:03 trong Số học

 

 

Bài 37. [AoPS] Với mỗi số nguyên $r$, chứng minh rằng tồn tại số tự nhiên $n_r$ sao cho với mọi số nguyên $n>n_r$ tồn tại ít nhất một số nguyên dương $k$ thoả mãn $1 \le k \le n-1$ và $p^r$ là ước của $\binom{n}{k}$ với $p$ là số nguyên tố nào đó.

Lời giải của mình.

Lấy $p,q$ là hai số nguyên tố bất kì.

Với mỗi $r$ chọn $n_r=max(p^r,q^r)+100$

Xét số nguyên $n$ bất kì lớn hơn $n_r$. Ta thấy luôn luôn có $p^a-1\neq q^b-1$ do $p\neq q$ nên $n$ ko thể có cả hai dạng $p^a-1$ hoặc là $q^a-1$

Giả sử $n\neq p^a-1$, khi đó biểu diển $p$-phân của $n$ sẽ có chữ số tận cùng khác $p-1$.

Ta sẽ sử dụng định lí $Kummer$

Giả sử $n=(b_tb_{t-1}...b_1b_0)_p$ với $b_0\neq p-1$ và do cách chọn $n_r$ nên $t>>r$.

CHọn $k=[b_rb_{r-1}...b_1(b_0+1)]_p$ 

Dễ thấy rằng khi trừ $n-k$ trong hệ $p$-phân sẽ được kết quả là một số có $r+1$ chữ số tận cùng bằng $p-1$ ( $p-1$ là chữ số lớn nhất trong hệ cơ số này)

Khi đó thì phép cộng $n-k$ với $k$ sẽ nhớ ít nhất là $r$ lần, theo định lí $Kummer$ ta có $v_p\binom{n}{k}\geq r$.

Ta có đpcm.

 

Bài 40.   Tìm tất cả các số tự nhiên $m$ sao cho với mọi sô thuộc tập $\{0,1,2,..,m-1\}$ đều đồng  dư với một số dạng $x^2+y^2$ mod $m$.




#637064 Marathon số học Olympic

Đã gửi bởi Visitor on 31-05-2016 - 10:38 trong Số học

Lời giải bài $30$ 
Chọn $n=16.a^{8}$ 
$n^2+n+1=(4a^4-2a^2+1)(4a^4+2a^2+1)(4a^4-4a^3+2a^2-2a+1)(4a^4+4a^3+2a^2+2a+1)$

Ta chỉ cần quan tâm đến việc chọn $4a^4+4a^3+2a^2+2a+1,4a^4+2a^2+1$ như thế nào là đúng đắn nhất 
Chọn $a \equiv 1 \pmod{7},a \equiv 1 \pmod{13}$ hay $a \equiv 1 \pmod{91}$ 
Khi đó $\sqrt{n}=4a^4>\frac{4a^4+4a^3+2a^2+2a+1}{13},\frac{4a^4+2a^2+1}{7}$ 
Như vậy các ước nguyên tố sau luôn sẽ bé hơn mấy số này 
P/s : Khá khó khăn trong việc chọn modulo 

Nếu đã phân tích được thế kia thì chọn modulo có khó khăn gì đâu em. Cho anh hỏi em biến đổi biểu thức kia ntn thế?   quá khủng




#636996 Marathon số học Olympic

Đã gửi bởi Visitor on 31-05-2016 - 01:24 trong Số học

Lời giải bài 29.

Giả sử tồn tại $P(x),Q(x)$

Do $lim \pi(x)=+\propto$ nên $degP>degQ$

Mà $lim \frac{\pi(x)}{x}=0\Rightarrow lim\frac{P(x)}{xQ(x)}=0\Rightarrow degP<degQ+1$
Mâu thuẫn. Vậy ta có đpcm.

Bài 30. Chứng minh rằng tồn tại vô số số tự nhiên $n$ sao cho tất cả ước nguyên tố của $n^2+n+1$ đều ko lớn hơn $\sqrt{n}$




#635905 Marathon Tổ hợp và rời rạc VMF

Đã gửi bởi Visitor on 27-05-2016 - 12:04 trong Tổ hợp và rời rạc

Lời giải bài 8.


 

a/ thay các lá bài bằng các số từ $1$ đến $13$ . Anne sẽ gọi lần lượt từ số $1$ tới số $13$, lặp lại một số vòng gọi như vậy. Giả sử ô trống nằm giữa $2$ số $i<j$ thì khi gọi đến $i$,số $i$ sẽ chuyển vào chỗ trống. Khi đó sẽ có 2 số cạnh ô trống là $i$ và một số $k$ nữa. Nếu $k>i$ thì khi gọi đến $k$ $k$ k sẽ di chuyển vào ô trống, và $i$ bh ko cạnh ô trống nữa, còn nếu $k<i$ thì $k$ sẽ dịch chuyển vào ô trống trong vòng gọi thứ 2. Tóm lại $i$ sẽ ko ở cạnh ô trống nữa và sẽ có số khác nhảy vào ô trống. Như vậy sau 1 số bước thì các quân bài ko còn ở vị trí ban đầu nữa.

b/Để quân Át, tức số $1$ ko năm cạnh ô trống thì Anne chỉ việc gọi như phần a/ nhưng sẽ bắt đầu gọi từ $2$ đến $13$. Số $1$ sẽ đứng im còn ô trống sẽ thay đổi nên sẽ có lúc nào đó số $1$ ko nằm cạnh ô trống.




#635751 Marathon số học Olympic

Đã gửi bởi Visitor on 26-05-2016 - 20:40 trong Số học

@Visitor: Có một chỗ chưa chuẩn là điều kiện phải là $\left\lfloor (i + 1)\alpha \right\rfloor \ge n + 1$, lúc này thì đoạn cuối không có vô lí

Dấu "=" xảy ra khi $(i+1)\alpha =n+1$, chỉ cần chọn $\alpha$ vô tỉ thì ko xảy ra được 




#635739 Marathon số học Olympic

Đã gửi bởi Visitor on 26-05-2016 - 19:40 trong Số học

Bài 9.

Hình như mình hiểu nhầm đề bài hay sao nhỉ @_@ mà lại chứng minh được là chia được

Giả sử ko chia được như đề bài tức là sẽ có số $n$ ko thuộc vào $S(\alpha ),S(\beta ),S(\gamma )$

Khi đó tồn tại $i$ sao cho $i\alpha <n$ và $(i+1)\alpha >n+1$ $\Rightarrow \frac{i}{n}< \frac{1}{\alpha }< \frac{i+1}{n+1}$

tương tự tồn tại $j$ : $\frac{j}{n}< \frac{1}{\alpha }< \frac{j+1}{n+1}$

Cộng vế ta được $\Rightarrow \frac{i+j}{n}< \frac{1}{\alpha }+\frac{1}{\beta  }< \frac{i+j+2}{n+1}$

chọn $ \frac{1}{\alpha }+\frac{1}{\beta  } = 1$ thì $ \frac{i+j}{n}<1< \frac{i+j+2}{n+1}\Rightarrow i+j<n<i+j+1$

Vô lí. Vậy mọi số tự nhiên đều thuộc $S(\alpha ),S(\beta )$, $\gamma$ tùy ý 

Chắc mình nhầm ở đâu @"@




#635671 Marathon số học Olympic

Đã gửi bởi Visitor on 26-05-2016 - 12:58 trong Số học

Lời giải bài 5:

Ta sử dụng bổ đề quen thuộc : $v_{p}((qp^k+r)!)=v_{p}(qp^k)+v_{p}(r)$ với $r<p^k$, $p$ là số nguyên tố, $q,r$ là số tự nhiên.

C/m: $v_{p}((qp^k+r)!)=\frac{qp^k+r-S_{p}(qp^k+r)}{p-1}$

Do $r<p^k$ nên $r$ ko thể có biểu diễn quá $k$ chữ số trong hệ cơ số $p$ nên $S_{p}(qp^k+r)= S_{p}(qp^k)+S_{p}(r)$

suy ra: $v_{p}((qp^k+r)!)=\frac{qp^k+r-S_{p}(qp^k)-S_{p}(r)}{p-1}= \frac{qp^k-S_{p}(qp^k)}{p-1}+\frac{r-S_{p}(r)}{p-1}=v_{p}((qp^k)!)-v_{p}(r!)$ (đpcm)

Trở lại bài toán: 

Xét khai triển một số $n$ bất kì $n=p_{1}^{a_1}p_{2}^{a_2}...p_{k}^{a_k}.Â$

Theo $mod d$ thì ta thấy sẽ có hữu hạn bộ số $(a_1,a_2,...a_k)$ , cụ thể là $d^k$ bộ. Vì thế khi $n$ tăng đến vô cùng thì sẽ có vô số bộ bị lặp lại theo $modd$.

Xét các số dạng $a_s=(p_1p_2...p_k)^s$, theo nhận xét trên thì sẽ tồn tại các số $n_1<n_2<...<n_d$ sao cho : 

$v_{p_t}((a_{n_i})!)\equiv v_{p_t}((a_{n_j})!)(mod.d)$ với mọi $t$ từ $1$ đến $k$ và mọi $1\leq i,j\leq d$.

Vì các $n_i$ có thể lớn tùy ý nên ta chọn sao cho các điều kiện của bổ đề được thỏa mãn:

$a_{n_1}<min(p_{t}^{n_2})$

$a_{n_1}+a_{n_2}<min(p_{t}^{n_3})$

$a_{n_1}+a_{n_2}+a_{n_3}<min(p_{t}^{n_4})$

$........$

Áp dụng bổ đề ta có với mọi $p_t$ thì: 

$v_{p_t}((a_{n_2}+a_{n_1})!)=v_{p_t}((a_{n_2})!)+v_{p_t}((a_{n_1})!) \equiv 2v_{p_t}((a_{n_1})!)   (mod .p_t)$

$v_{p_t}((a_{n_3}+a_{n_2}+a_{n_1})!)=v_{p_t}((a_{n_3})!)+v_{p_t}((a_{n_2}+a_{n_1})!)\equiv 3v_{p_t}((a_{n_1})!) $

$........$

$v_{p_t}((a_{n_d}+...+a_{n_2}+a_{n_1})!)\equiv dv_{p_t}((a_{n_1})!) \equiv0$

CHọn $n=a_{n_d}+...+a_{n_2}+a_{n_1}$ thì $n$ thỏa đề. 
Vì $n_i$ lớn tùy ý nên cũng sẽ có vô số $n$ thỏa mãn đề bài.đpcm.

#

CHo mình nợ bài 6 vài phút.

Chả biết chọn bài nào, chọn đại 1 bài vui vui vậy :)

 

 

Bài toán 6. ( Toán cổ) Tìm tất cả số tự nhiên $n$ để có thể phân hoạch tập $S=\{1,2,3...,4n\}$ thành các tập con $4$ phần tử sao cho trong mỗi tập tồn tại một phần tử bằng trung bình cộng của ba phần tử còn lại.




#635226 $\left ( a+n,b+n \right )=1$

Đã gửi bởi Visitor on 24-05-2016 - 18:32 trong Số học

1, $a,b\in \mathbb{N}*$. Chứng minh rằng có vô số $n\in N*$ sao cho    $\left ( a+n,b+n \right )=1$  

                                           

Chọn $n$ để $n+a$ là sô nguyên tố lớn hơn trị tuyệt đối $a-b$ 

khi đó $(a+n,b+n)=(a+n,a-b)=1$.




#631745 ĐỀ THI OLYMPIC CHUYÊN KHOA HỌC TỰ NHIÊN 2016

Đã gửi bởi Visitor on 07-05-2016 - 14:23 trong Thi HSG cấp Tỉnh, Thành phố. Olympic 30-4. Đề thi và kiểm tra đội tuyển các cấp.

 

ĐỀ THI OLYMPIC CHUYÊN KHOA HỌC TỰ NHIÊN 2016

Ngày 1 (07/05/2016)
 

Câu 4. Cho dãy số $(a_{n})_{n\in\mathbb{Z}^{+}}$ xác định như sau $$\begin{cases}a_{1} = 0 \\ a_{2} = 1 \\ a_{2n} = 2a_{n} + 1 \\ a_{2n + 1} = 2a_{n}\end{cases}$$ với mọi $n \in \mathbb{Z}^{+}$. Chứng minh rằng tồn tại vô số số nguyên dương $k$ sao cho $a_{k} = 2016$ và tìm số nguyên dương $k$ nhỏ nhất thỏa mãn điều này.
 

Câu này vui. CHuyển sang hệ cơ số 2 thì ta thấy: $a_{2n}$ sẽ bằng $a_n$ thêm số $1$ ở cuối, còn $a_{2n+1}$ bằng $a_n$ thêm số $0$ ở cuối. Mà $2$ số hạng bắt đầu là $0,1$ nên cứ thêm $1$ và $0$ vào cuối như vậy ta sẽ có mọi số tự nhiên :v

Còn $2016=(11111100000)_2$, cứ bỏ lần lượt các chữ số cuối đi là tìm được :v

 À còn vô hạn thì suy ra trực tiếp từ chuyện có vô hạn số $0$




#631740 $x_{1}^{n_{1}} + x_{2}^{n_...

Đã gửi bởi Visitor on 07-05-2016 - 13:33 trong Số học

Cho $n_{1}, n_{2}, \cdots , n_{k + 1}$ là các số nguyên dương sao cho $\gcd(n_{k + 1}, n_{i}) = 1 \; \forall i < k + 1$. Chứng minh rằng tồn tại vô số bộ số nguyên dương$ x_{i}$ sao cho $x_{1}^{n_{1}} + x_{2}^{n_{2}} + \cdots x_{k}^{n_{k}} = x_{k + 1}^{n_{k + 1}}$.

Số mũ lung tung nên đưa về bằng nhau cho dễ.

Đặt $A=n_{1}n_{2}...n_{k}$. CHọn $x_i=k^{l.\frac{A}{n_{i}}}$ , $:$ nào đó.

suy ra $x_i^{n_i}=k^{lA}\Rightarrow x_1^{n_1}+x_2^{n_2}+...+x_k^{n_k}=k^{lA+1}$

chỉ việc chọn $l$ sao cho $lA+1 \vdots n_{k+1}$ là ta sẽ có được $x_{k+1}=k^{\frac{lA+1}{n_{k+1}}}$.

Mà tất nhiên là có vô số $l$ như vậy nên cũng có vố số bộ tm.




#631688 Có tồn tại vô hạn hay không các hợp số có tính chất $L$?

Đã gửi bởi Visitor on 07-05-2016 - 00:42 trong Số học

Một số nguyên dương $n$ được gọi là có tính chất $L$ nếu với mọi số nguyên dương $a$ tùy ý, thì mệnh đề sau luôn đúng: 'Nếu $n\mid a^{n} - 1$ thì $n^{2}\mid a^{n} - 1$'. Hỏi có tồn tại vô hạn hay không các hợp số có tính chất $L$?

Dễ thấy nếu $m,n$ thỏa mãn mà $gcd(m,n)=1$ thì $mn$ cũng thỏa.    (1)

Từ nhận xét trên ta thấy rằng có vẻ như sẽ có vô hạn :v nếu thế thì ta chỉ cần có vô số nguyên tố đều có $L$  (2)

thật vậy: gọi $p$ là 1 số nguyên tố . Giả sử : $p|a^p-1\Rightarrow p|a-1$ ( Cô si)

$\Rightarrow a^p-1= (a-1)(a^{p-1}+a^{p-2}+...+a+1) \vdots p^2$ ( do $a\equiv 1(modp)$ )

suy ra (2) đúng. Từ (1) và (2) ta có đpcm :v

 

Dạo này học hình hơi nhiều -.-




#626717 Chứng minh rằng $v_{p}(\frac{x^{k} + y^...

Đã gửi bởi Visitor on 12-04-2016 - 11:47 trong Số học

Cho $x, y$ là hai số nguyên dương nguyên tố cùng nhau, $p \ge 3$ là một số nguyên tố, và $k$ là một số nguyên dương nguyên tố cùng nhau với $p - 1$.
Chứng minh rằng $v_{p}(\frac{x^{k} + y^{k}}{x + y}) = v_{p}(k)$

Nguồn: AoPS

Dựa vào LTE ta chỉ cần chứng minh nếu $x^k+y^k \vdots p$ thì $x+y \vdots p$

Thật vậy, do $(k,p-1)=1$ nên là $k$ lẻ và tồn tại $u,v$ ko âm sao cho $uk-v(p-1)=1$

Ta có : $x^{k}\equiv -y^k\equiv (-y)^{k}(modp)\Rightarrow x^{uk} \equiv(-y)^{uk}(modp)$

và do $(x,p)=(y,p)=1$ : $x^{p-1}\equiv 1\equiv (-y)^{p-1}(modp)\Rightarrow x^{v(p-1)} \equiv(-y)^{v(p-1)}(modp)$

suy ra $x \equiv -y(modp)$ $\Rightarrow x+y \vdots p$ $đpcm$




#626685 Chứng minh rằng tồn tại số nguyên dương $m$ thỏa cả hai điều kiện

Đã gửi bởi Visitor on 11-04-2016 - 22:54 trong Số học

$n > 10000$ là một số tự nhiên cho trước. Chứng minh rằng tồn tại số nguyên dương $m$ thỏa cả hai điều kiện:
i) $m$ viết được dưới dạng tổng hai số chính phương
ii) $0 < m - n < 3\sqrt[4]{n}$

Nguồn: thành viên LichKing - mathscope.

chọn $m-n=4k+1$ là $OK$ :v

dựa vào nhận xét $2$ ở đây http://diendantoanho...p-và-2ab-c2-d1/




#626680 $$\lim_{n\to \infty}S(2^{n}) = +...

Đã gửi bởi Visitor on 11-04-2016 - 22:43 trong Số học

Ký hiệu $S(n)$ là tổng các chữ số của $n$ trong hệ thập phân. Chứng minh rằng
$$\lim_{n\to \infty}S(2^{n}) = +\infty$$

Nguồn: Facebook

Ta làm kiểu chia các chữ số của $2^n$ thành các nhóm nhỏ.

Giả sử $2^n=(a_{k}a_{k-1}...a_{1})_{(10)}$. Ta chứng minh với $i$ thíc hợp thì trong các chữ số $a_{i+1},a_{i+2},...a_{4i}$ sẽ có ít nhất một chữ số khác $0$.

Thật vậy, phản chứng là ko xảy ra điều trên, tức là  các chữ số $a_{i+1},a_{i+2},...a_{4i}$ đều bằng $0$.

 Đặt $a=(a_{i}a_{i-1}...a_{1})_{(10)}$ suy ra $2^n-a \vdots 10^{4i} \vdots2^{4i}$.

CHọn $i<\frac{n}{4}$ thì ta sẽ có $a \vdots 2^{4i}$, vô lí vì $a < 10^i <2^{4i}$

vậy nx trên đúng.

Với $n$ càng lớn thì càng chia được nhiều nhóm kiểu như $a_{i+1},a_{i+2},...a_{4i}$, mà tổng các chữ số mỗi nhóm khác $0$ nên hiển nhiên $\lim_{n\to \infty}S(2^{n}) = +\infty$




#626676 Cho $p\in \mathbb{P}$. Chứng minh luôn tồn tại...

Đã gửi bởi Visitor on 11-04-2016 - 22:29 trong Số học

Cho $p\in \mathbb{P}$. Chứng minh luôn tồn tại $a, b \in \mathbb{Z}^{+}$ thoả $a + b = p$ và $2ab = c^{2} + d^{2}$

Nhận xét

Hướng đi của mình dài :v

Nhận xét $1$: mọi số nguyên tố dạng $4l+1$ đều có thể biểu diễn dưới dạng tổng $2$ số chính phương. Cái này cơ bản.

Nhận xét $2$: mọi số nguyên dương có dạng $2^{\alpha }(4l+1)$ với $\alpha  =0$ hoặc $\alpha  =1$ hoặc $\alpha=2$ hoặc $\alpha$ bất kì :))

đều có thể biểu diễn dưới dạng tổng $2$ số chính phương

cái này cm rất dễ dựa vào nhận xét $1$.

 

Nếu $p=4t+1$, chọn $a=4$, $b=4t-3$ thì $2ab=2^3(4t-3)$, theo nhận xét $2$ ta có $đpcm$

Nếu $p=4t+3$, chọn $a=2$, $b=4t+1$ thì $2ab=2^2(4t+1)$, theo nhận xét $2$ ta cũng có $đpcm$.

Các trường hợp $p$ nhỏ nhỏ rất dễ để chỉ ra nên ko xét.




#623334 Hỏi khi đó $a^{2017} - a$ được tô màu gì?

Đã gửi bởi Visitor on 29-03-2016 - 00:04 trong Số học




#623327 Hỏi khi đó $a^{2017} - a$ được tô màu gì?

Đã gửi bởi Visitor on 28-03-2016 - 23:27 trong Số học

Tập hợp các số nguyên dương được tô bởi 2 màu đen và trắng thỏa các điều kiện:
i) Tổng hai số khác màu thì được tô bằng màu đen.
ii) Có vô hạn số được tô bằng màu trắng.
Giả sử $a \ge 2$ là một số nguyên dương được tô màu trắng. Hỏi khi đó $a^{2017} - a$ được tô màu gì?

Spoiler

Minh cũng thấy hứng thú :v

Ta thấy:

+Nếu số $1$ màu trắng thì tất cả các số đều trắng vì nếu có số $n$ màu đen thì tất cả các số sau $n$ đều đen, mâu thuẫn với $ii$

Vậy $a^{2017}-a$ màu trắng.

+Nếu số $1$ màu đen. Ta gọi $k+1$ là số đầu tiên được tô màu trắng, tức là $1,2,...k$ đều màu đen.Từ $i$ ta có được là tất cả các số không có dạng $t(k+1)$ sẽ có màu đen. Ta chứng minh tất cả các số dạng $t(k+1)$ đều màu trắng.

Thật vậy, có $k+1$ trắng nên nếu mà có $h(k+1)$ nào đó mà đen thì theo $i$ : $t(k+1)$ sẽ đen với mọi $t\geq h$. Mâu thuẫn với $ii$.

Vậy chỉ có các số là bội của $k+1$ mới trắng còn lại đen hết.

Do đó, vì $a^{2017}-a\vdots a$ nên hiển nhiên nó màu trắng :v




#621464 Chứng minh rằng $\dbinom{2n}{n}$ chia hết...

Đã gửi bởi Visitor on 20-03-2016 - 20:29 trong Số học

Bài này số mũ bt cũng ra nhưng chắc dài. cho mình xin tài liệu về $Kummer$ đc ko Jinbe. Trước t cũng có của viện toán nhưng lại chả nhớ để đâu :))



#620872 $x^{2}-y^{2}=y^{2}-z^{2}=z^...

Đã gửi bởi Visitor on 18-03-2016 - 08:12 trong Số học

Tìm các số nguyên dương $x,y,z,t$ đôi một phân biệt thỏa $x^{2}-y^{2}=y^{2}-z^{2}=z^{2}-t^{2}.$

Bài toán kinh điển từ năm $1640$: ko có cấp số cộng nào có bốn số hạng liên tiếp là số chính phương. 

và lời giải hay nhất của nó:

"

No Four Squares In Arithmetic Progression

 

To prove that four consecutive terms in an arithmetic sequence cannot all be squares, suppose there exist four squares A2, B2, C2, D2 in increasing arithmetic progression, i.e., we have B2-A2 = C2-B2 = D2-C2. We can assume the squares are mutually co-prime, and the parity of the equation shows that each square must be odd. Hence we have co-prime integers u,v such that A = u-v, C = u+v, u2+v2 = B2, and the common difference of the progression is (C2-A2)/2 = 2uv.

 

We also have D2 - B2 = 4uv, which factors as [(D+B)/2][(D-B)/2] = uv. The two factors on the left are co-prime, as are u and v, so there exist four mutually co-prime integers a,b,c,d (exactly one even) such that u = ab, v = cd, D+B = 2ac, and D-B = 2bd. This implies B = ac-bd, so we can substitute into the equation u2+v2 = B2  to give (ab)2+(cd)2 = (ac-bd)2. This quadratic is symmetrical in the four variables, so we can assume c is even and a, b, d are odd. From this quadratic equation we find that c is a rational function of the square root of  a4-a2d2+d4, which implies there is an odd integer m such that  a4-a2d2+d4= m2.

 

Since a and d are odd there exist co-prime integers x and y such that a2 = k(x+y) and d2 = k(x-y), where k = ±1. Substituting into the above quartic gives x2+3y2 = m2, from which it's clear that y must be even and x odd. Changing the sign of x if necessary to make m+x divisible by 3, we have 3(y/2)2 = [(m+x)/2][(m-x)/2], which implies that (m+x)/2 is three times a square, and (m-x)/2 is a square. Thus we have co-prime integers r and s (one odd and one even) such that (m+x)/2 = 3r2, (m-x)/2 = s2, m = 3r2+s2, x = 3r2-s2, and y = ±2rs.

 

Substituting for x and y back into the expressions for a2 and d2 (and transposing if necessary) gives a2 = k(s+r)(s-3r) and d2 = k(s-r)(s+3r). Since the right hand factors are co-prime, it follows that the four quantities (s-3r), (s-r), (s+r), (s+3r) must each have square absolute values, with a common difference of 2r. These quantities must all have the same sign, because otherwise the sum of two odd squares would equal the difference of two odd squares, i.e., 1+1 = 1-1 (mod 4), which is false.

 

Therefore, we must have |3r| < s, so from m = 3r2 + s2 we have 12r2 < m. Also the quartic equation implies m < a2 + d2, so we have the inequality |2r| < |SQRT(2/3)max(a,d)|. Thus we have four squares in arithmetic progression with the common difference |2r| < |2abcd|, the latter being the common difference of the original four squares. This contradicts the fact that there must be a smallest absolute common difference for four squares in arithmetic progression, so the proof is complete.□ "




#619426 Chứng minh tồn tại 3 điểm là ba đỉnh của 1 tam giác có tung và hoành độ của t...

Đã gửi bởi Visitor on 09-03-2016 - 21:55 trong Tổ hợp và rời rạc

bạn ơi.. điều quan trọng là cả tung lẫn hoành đều chia hết cho 3... vì vậy mà bạn cần xét kỹ hơn..bạn nên xem lại

bạn ơi... mình xét $(i,j)$ là đã xét cả tung hoành độ rồi mà...




#619424 $u_{n}$ không nguyên tố cùng nhau với $u_{n+1...

Đã gửi bởi Visitor on 09-03-2016 - 21:45 trong Dãy số - Giới hạn

Cho dãy số nguyên dương $(u_{n})$ thoả $u_{1}\neq1$ và $u_{n+1}$ là số nhỏ nhất không nguyên tố cùng nhau với $u_{n}$ mà khác $u_{1},u_{2},...,u_{n}.$ Chứng minh mọi số nguyên dương khác $1$ đều xuất hiện trong $(u_{n}).$

Bài này hay thật

+,Đầu tiên ta đi chứng minh dãy chứa vô hạn số nguyên tố.

Thật vậy, giả sử chỉ có hữu hạn số nguyên tố thuộc dãy là $p_1,p_2,...p_n$,$p_1=min(p_i)$

Khi đó tất cả các số thuộc dãy $u_n$ chỉ có các ước nguyên tố là các số trên, tức là có dạng $p_{1}^{\alpha _1}p_{2}^{\alpha _2}...p_{n}^{\alpha _n}$

mà dãy $u_n$ là dãy vô hạn nên đến một lúc nào đó sẽ có $1$ số $k$ để mà $p_{1}^{k}> q.p_1p_2...p_n$ , $q\neq p_i$

do đó $ q.p_1p_2...p_j$ sẽ thuộc dãy suy ra $q$ cũng thuộc dãy này, vô lí.

+,Tiếp theo ta chứng minh mọi số chẵn $2k$ thuộc dãy bằng qui nạp.

Dễ thấy $2,4 \in u_n$

Giả sử đúng đến $k=t$ ta chứng minh đúng với $k=t+1$. Do nhận xét đầu tiên nên sẽ có số nguyên tố $q \in u_n$ mà $q>2(t+1)$

Đặt $q= u_m$. Nếu tồn tại $1\leq h\leq m-1$ sao cho $u_h=2q$, mà $q>2(t+1)$ suy ra $u_{h+1}=2(t+1)$

Nếu ko tồn tại $u_h$ như ở trên thì $u_{m+1}=2q\Rightarrow u_{m+1}=2(t+1)$

Vậy qui nạp xong.

+,Cuối cùng ta qui nạp mọi số nguyên dương đều xuất hiện trong $u_n$

bước cơ sở bỏ qua. 

giả sử đúng đến $t$ ta cm đúng vs $t+1$, thật vậy : lấy ra $a_l=2(t+1)$, mà mọi số từ $2$ đến $t$ đều xuất hiện trong $u_n$ rồi nên mọi ước của $t+1$ đã xuất hiện , do đó $a_{l+1}=t+1$

+, kết thúc 




#619328 $\lim_{n \to \infty }(\frac{1+2+3+..,...

Đã gửi bởi Visitor on 09-03-2016 - 14:19 trong Dãy số - Giới hạn

bài $3$ :  do tử số bị chặn nên giới hạn bằng $0$




#619308 Tồn tại hay không tam giác có 3 đỉnh khác màu

Đã gửi bởi Visitor on 09-03-2016 - 12:24 trong Tổ hợp và rời rạc

Cho một tam giác có 3 đỉnh được tô các màu là đỏ, vàng, xanh. Trong tam giác đó lấy các điểm bất kì cũng được tô một trong 3 màu trên. Nối các điểm đó với nhau và nối với các đỉnh của tam giác để tạo ra các tam giác mới sao cho không có tam giác nào chồng chéo lên nhau. Có tồn tại hay không một tam giác khác tam giác ban đầu có 3 đỉnh tô 3 màu khác nhau?

Không chồng chéo thì chỉ có trường hợp có $1$ điểm thôi chứ nhỉ. Mà $1$ điểm thì có đpcm 




#619306 Chứng minh tồn tại 3 điểm là ba đỉnh của 1 tam giác có tung và hoành độ của t...

Đã gửi bởi Visitor on 09-03-2016 - 12:18 trong Tổ hợp và rời rạc

trong mặt phẳng tọa độ Oxy, cho 19 điểm có hành độ và tung độ là các số nguyên, trong 19 điểm đó không có ba điểm nào thẳng hàng. chứng minh trong 19 điểm trên có ít nhất 3 điểm là 3 đỉnh của một tam giác có hoành độ và tung độ của trong tâm tam giác đó là các số nguyên..

Xét các điểm $(i,j)$ với $i,j \in  \left \{ 0,1,2 \right \}$ .Có tất cả $3*3=9$ điểm. Mà có $19$ điểm nên sẽ có ít nhất $3$ điểm cùng số dư với $1$ điểm $(i,j)$ khi chia cho $3$. $đpcm$